LSAT and Law School Admissions Forum

Get expert LSAT preparation and law school admissions advice from PowerScore Test Preparation.

User avatar
 katehos
PowerScore Staff
  • PowerScore Staff
  • Posts: 184
  • Joined: Mar 31, 2022
|
#96023
Hi rrurrec!

Yes, it's certainly possible that (E) could weaken the argument -- hence why we can eliminate it! You can, of course, alter the actual sizes of the dissatisfied and satisfied groups to create a scenario in which (E) strengthens the argument but that still leaves (E) as an incorrect answer since it could weaken, could strengthen, or could even do nothing. Ultimately, (E) does not definitively strengthen the argument, and, like you mentioned, likely weakens the argument because it points out flaws with the sample. Good work!

Hope this helps :)
Kate

Get the most out of your LSAT Prep Plus subscription.

Analyze and track your performance with our Testing and Analytics Package.